Does pattern always true?

Algebra Level 4

Let p ( x ) p(x) be a polynomial of degree 4 with real coefficients satisfying p ( 0 ) = 0 p(0)=0 , p ( 1 ) = 1 2 p(1)=\dfrac{1}{2} , p ( 2 ) = 2 3 p(2)=\dfrac{2}{3} , p ( 3 ) = 3 4 p(3)=\dfrac{3}{4} and p ( 4 ) = 4 5 p(4)=\dfrac{4}{5} . Evaluate the value of p ( 5 ) p(5) . Enter your answer with 3 significant figures.


The answer is 0.667.

This section requires Javascript.
You are seeing this because something didn't load right. We suggest you, (a) try refreshing the page, (b) enabling javascript if it is disabled on your browser and, finally, (c) loading the non-javascript version of this page . We're sorry about the hassle.

4 solutions

Raymond Chan
Nov 18, 2017

Define q ( x ) = ( x + 1 ) p ( x ) x q(x)=(x+1)p(x)-x

Note that for all n=0, 1, 2, 3, 4,

p ( n ) = n n + 1 p(n)=\frac{n}{n+1}

Therefore, q ( n ) = ( n + 1 ) n n + 1 n = n n = 0 q(n)=(n+1)\frac{n}{n+1}-n=n-n=0

So, the polynomial q(x) has roots 0, 1, 2, 3, 4

As a result, we can rewrite q(x) into the form K x ( x 1 ) ( x 2 ) ( x 3 ) ( x 4 ) Kx(x-1)(x-2)(x-3)(x-4)

where K is a real constant.

Recall, by definition, that q ( x ) = ( x + 1 ) p ( x ) x q(x)=(x+1)p(x)-x

Therefore, ( x + 1 ) p ( x ) x = K x ( x 1 ) ( x 2 ) ( x 3 ) ( x 4 ) (x+1)p(x)-x=Kx(x-1)(x-2)(x-3)(x-4)

To solve for K, simply put x=-1 to the identity.

Now we have ( 1 + 1 ) p ( 1 ) ( 1 ) = K ( 1 ) ( 1 1 ) ( 1 2 ) ( 1 3 ) ( 1 4 ) (-1+1)p(-1)-(-1)=K(-1)(-1-1)(-1-2)(-1-3)(-1-4)

Simplify, we have 1 = 120 K 1=-120K

K = 1 120 K=\frac{-1}{120}

Thus, we have q ( x ) = 1 120 x ( x 1 ) ( x 2 ) ( x 3 ) ( x 4 ) q(x)=\frac{-1}{120}x(x-1)(x-2)(x-3)(x-4)

Put x=5 into the expression, we have q ( 5 ) = 1 120 ( 5 ) ( 5 1 ) ( 5 2 ) ( 5 3 ) ( 5 4 ) = 1 q(5)=\frac{-1}{120}(5)(5-1)(5-2)(5-3)(5-4)=-1

By the definition of q(x), q ( 5 ) = 6 p ( 5 ) 5 q(5)=6p(5)-5

Therefore, p ( 5 ) = q ( 5 ) + 5 6 = 1 + 5 6 = 2 3 = 0.667 p(5)=\frac{q(5)+5}{6}=\frac{-1+5}{6}=\frac{2}{3}=\boxed{0.667}

For those of you who say the answer is 5 6 \frac{5}{6} , you are wrong because p(x) is a polynomial, not a sequence!

The question is come from Hong Kong Advanced Level Examination 2006 Pure Mathematics Paper 1 question 3

My friend in Hong Kong did't know how to solve this and sent the picture to me. It is pretty cool so I cancel out some part of the question and post it here.

There's a typo in line 5. It should be "q(x) = (x+1)p(x)-x".

Nikhil N - 3 years, 5 months ago

Done in same way...🖒🖒upvoted.

rajdeep brahma - 3 years, 4 months ago

A standard way of solving this type of problem is as follows:

\begin{aligned} p(x) & = \sum_{k=0}^4 a_k \prod_{\substack{j=0 \\ j \ne k}}^4 (x-j) & \small \color{#3D99F6} \text{where }a_k \text{ is a coefficient.} \end{aligned} = a 0 ( x 1 ) ( x 2 ) ( x 3 ) ( x 4 ) No ( x 0 ) + a 1 x ( x 2 ) ( x 3 ) ( x 4 ) No ( x 1 ) + a 2 x ( x 1 ) ( x 3 ) ( x 4 ) No ( x 2 ) + a 3 x ( x 1 ) ( x 2 ) ( x 4 ) No ( x 3 ) + a 4 x ( x 1 ) ( x 2 ) ( x 3 ) No ( x 4 ) \quad \ \ = \underbrace{a_0(x-1)(x-2)(x-3)(x-4)}_{\color{#3D99F6}\text{No }(x-0)} + \underbrace{a_1x(x-2)(x-3)(x-4)}_{\color{#3D99F6}\text{No }(x-1)} + \underbrace{a_2x(x-1)(x-3)(x-4)}_{\color{#3D99F6}\text{No }(x-2)} + \underbrace{a_3x(x-1)(x-2)(x-4)}_{\color{#3D99F6}\text{No }(x-3)} + \underbrace{a_4x(x-1)(x-2)(x-3)}_{\color{#3D99F6}\text{No }(x-4)} p ( 0 ) = a 0 ( 1 ) ( 2 ) ( 3 ) ( 4 ) = 0 a 0 = 0 p ( 1 ) = a 1 ( 1 ) ( 1 ) ( 2 ) ( 3 ) = 1 2 a 1 = 1 12 p ( 2 ) = a 2 ( 2 ) ( 1 ) ( 1 ) ( 2 ) = 2 3 a 2 = 1 6 p ( 3 ) = a 3 ( 3 ) ( 2 ) ( 1 ) ( 1 ) = 3 4 a 2 = 1 8 p ( 4 ) = a 4 ( 4 ) ( 3 ) ( 2 ) ( 1 ) = 4 5 a 2 = 1 30 \begin{aligned} p(0) & = a_0(-1)(-2)(-3)(-4) = 0 & \small \color{#3D99F6} \implies a_0 = 0 \\ p(1) & = a_1(1)(-1)(-2)(-3) = \frac 12 & \small \color{#3D99F6} \implies a_1 = -\frac 1{12} \\ p(2) & = a_2(2)(1)(-1)(-2) = \frac 23 & \small \color{#3D99F6} \implies a_2 = \frac 16 \\ p(3) & = a_3(3)(2)(1)(-1) = \frac 34 & \small \color{#3D99F6} \implies a_2 = -\frac 18 \\ p(4) & = a_4(4)(3)(2)(1) = \frac 45 & \small \color{#3D99F6} \implies a_2 = -\frac 1{30} \end{aligned}

Therefore, we have:

p ( x ) = x ( x 2 ) ( x 3 ) ( x 4 ) 12 + x ( x 1 ) ( x 3 ) ( x 4 ) 6 x ( x 1 ) ( x 2 ) ( x 4 ) 8 + x ( x 1 ) ( x 2 ) ( x 3 ) 30 p ( 5 ) = 5 ( 3 ) ( 2 ) ( 1 ) 12 + 5 ( 4 ) ( 2 ) ( 1 ) 6 5 ( 4 ) ( 3 ) ( 1 ) 8 + 5 ( 4 ) ( 3 ) ( 2 ) 30 = 5 2 + 20 3 15 2 + 4 = 2 3 0.667 \begin{aligned} p(x) & = - \frac {x(x-2)(x-3)(x-4)}{12} + \frac {x(x-1)(x-3)(x-4)}6 - \frac {x(x-1)(x-2)(x-4)}8 + \frac {x(x-1)(x-2)(x-3)}{30} \\ \implies p(5) & = - \frac {5(3)(2)(1)}{12} + \frac {5(4)(2)(1)}6 - \frac {5(4)(3)(1)}8 + \frac {5(4)(3)(2)}{30} \\ & = - \frac 52 + \frac {20}3 - \frac {15}2 + 4 \\ & = \frac 23 \approx \boxed{0.667} \end{aligned}

That is a little bashy .but a general strategy...will work most of times

rajdeep brahma - 3 years, 4 months ago
David Vreken
Dec 8, 2017

A polynomial of degree 4 with real coefficients has the equation y = a x 4 + b x 3 + c x 2 + d x + e y = ax^4 + bx^3 + cx^2 + dx + e .

When p ( 0 ) = 0 p(0) = 0 , 0 = a ( 0 ) 4 + b ( 0 ) 3 + c ( 0 ) 2 + d ( 0 ) + e 0 = a(0)^4 + b(0)^3 + c(0)^2 + d(0) + e , so e = 0 e = 0 , and y = a x 4 + b x 3 + c x 2 + d x y = ax^4 + bx^3 + cx^2 + dx .

When p ( 1 ) = 1 2 p(1) = \frac{1}{2} , 1 2 = a ( 1 ) 4 + b ( 1 ) 3 + c ( 1 ) 2 + d ( 1 ) \frac{1}{2} = a(1)^4 + b(1)^3 + c(1)^2 + d(1) or 2 a + 2 b + 2 c + 2 d = 1 2a + 2b + 2c + 2d = 1 .

When p ( 2 ) = 2 3 p(2) = \frac{2}{3} , 2 3 = a ( 2 ) 4 + b ( 2 ) 3 + c ( 2 ) 2 + d ( 2 ) \frac{2}{3} = a(2)^4 + b(2)^3 + c(2)^2 + d(2) or 24 a + 12 b + 6 c + 3 d = 1 24a + 12b + 6c + 3d = 1 .

When p ( 3 ) = 3 4 p(3) = \frac{3}{4} , 3 4 = a ( 3 ) 4 + b ( 3 ) 3 + c ( 3 ) 2 + d ( 3 ) \frac{3}{4} = a(3)^4 + b(3)^3 + c(3)^2 + d(3) or 108 a + 36 b + 12 c + 4 d = 1 108a + 36b + 12c + 4d = 1 .

When p ( 4 ) = 4 5 p(4) = \frac{4}{5} , 4 5 = a ( 4 ) 4 + b ( 4 ) 3 + c ( 4 ) 2 + d ( 4 ) \frac{4}{5} = a(4)^4 + b(4)^3 + c(4)^2 + d(4) or 320 a + 80 b + 20 c + 5 d = 1 320a + 80b + 20c + 5d = 1 .

Solving this system of equations gives a = 1 120 a = -\frac{1}{120} , b = 11 120 b = \frac{11}{120} , c = 23 60 c = -\frac{23}{60} , and d = 4 5 d = \frac{4}{5} .

So p ( x ) = 1 120 x 4 + 11 120 x 3 23 60 x 2 + 4 5 x p(x) = -\frac{1}{120}x^4 + \frac{11}{120}x^3 - \frac{23}{60}x^2 + \frac{4}{5}x , which means p ( 5 ) = 1 120 ( 5 ) 4 + 11 120 ( 5 ) 3 23 60 ( 5 ) 2 + 4 5 ( 5 ) = 2 3 0.667 p(5) = -\frac{1}{120}(5)^4 + \frac{11}{120}(5)^3 - \frac{23}{60}(5)^2 + \frac{4}{5}(5) = \frac{2}{3} \approx 0.667 .

Sibasish Mishra
Dec 21, 2017

What is d1 d2 or d3??

rajdeep brahma - 3 years, 4 months ago

0 pending reports

×

Problem Loading...

Note Loading...

Set Loading...